Symmetriefaktor für Feynman-Diagramme in der ϕ4ϕ4\phi^4-Theorie für nnn-Punkte Grüne Funktion

Ich arbeite mit zwei Theorien.

Theorie A: H ich N T = D 3 X M G 2 ϕ φ 2

Theorie B: ϕ 4 -Interaktion: H ich N T = D 3 X λ 4 ! ϕ ( X ) 4

Wo M ist die dem Feld zugeordnete Masse ϕ .

Ich muss die berechnen N -Punkte Green-Funktion auf Baumebene für diese Theorien, G A ( P 1 , , P N ) , G B ( P 1 , , P N ) , unter Berücksichtigung der Grenze M + für die Streuung von Skalarteilchen von φ Feld

In allen folgenden Diagrammen v ist die Gesamtzahl der Scheitelpunkte und der externen Propagatoren (durch die Pfeile angegeben). N .

Feynman-Diagramm für Theorie A

feynA

Lassen Sie uns mit der Theorie A arbeiten und versuchen, den Symmetriefaktor dieses Diagramms zu berechnen. Wenn wir haben N also externe Propagatoren N = v + 2 . Die Anzahl der Propagatoren des Feldes ϕ Ist # Δ ϕ = v / 2 , während die Zahl der Δ φ Ist # Δ φ = v / 2 1 und so ist die Gesamtzahl der Propagatoren # Δ = v 1 = N 3 . Ich berechne den Symmetriefaktor als

1 S ! = 1 2 v 2 2 3 # Δ φ 2 # Δ φ = 1 2 v 2 2 6 v 2 1 = ( 3 2 ) v 2 1

weil die Anfangs- und Endknoten 2 mögliche gleiche Konfigurationen haben (Punkt P 1 kann auf zwei verschiedene Arten mit dem anfänglichen Scheitelpunkt kontrahiert werden, dasselbe gilt für P N ) und tragen dann mit a bei 2 2 Faktor. Dann für jeden internen φ -Propagator haben wir 3 2 Möglichkeiten, externe Impulse und Scheitelbeine zu kontrahieren, um ein solches Diagramm zu erhalten.

Ist es wahr oder übersehe ich etwas?

Feynman-Diagramm für Theorie B

feynB

In diesem Fall N = 2 ( v + 1 ) , # Δ φ = v 1 = ( N 4 ) / 2 .

ich würde sagen 1 / S ! = 1 denn für jeden Knoten gibt es 4 ! mögliche Wege, Beine und externe Impulse zu kontrahieren und so,

1 S ! = 1 ( 4 ! ) v ( 4 ! ) v = 1

Aber ich bin mir nicht sicher, ob das richtig ist ... Was ist der Symmetriefaktor eines solchen Diagramms? Wie leitet man es ab?

Können Sie die ersten paar Fälle (low v ) ausdrücklich? Dies sollte entweder Ihre Formel widerlegen oder Ihnen die Gewissheit geben, dass sie richtig ist.

Antworten (1)

Der Symmetriefaktor sollte in beiden Theorien 1 sein. Zunächst einige Bemerkungen. Wenn Sie amputierte Diagramme berechnen, werden die externen Propagatoren aus der Auswertung der Amplitude entfernt, es ist jedoch (für mich) unnatürlich, sie bei der Berücksichtigung von Symmetriefaktoren aus der Zählung zu entfernen. Theoretisch würde KI das sagen # Δ φ = 3 v 2 + 1 und in Theorie B das # Δ φ = 3 v + 1 . Übrigens in Theorie A N Ist v + 2 und nicht v 2 .

Berechnung der Dochtkontraktion in Theorie A:

Der Symmetriefaktor ist 1 v ! 1 2 v C Wo C ist die Anzahl der Dochtkontraktionen, die die gewünschte Form erzeugen. Sie müssen wählen, an welchen internen Scheitelpunkt angefügt wird P 1 , P 2 (gibt einen Faktor an v ) dann welcher Scheitelpunkt an den vorherigen anschließt usw. Sie erhalten also einen Faktor von v ! . Dann kontrahieren Sie die ϕ 's, für die es keine Auswahl gibt. Schließlich schließen Sie den Vertrag ab φ 'S. Die äußersten Eckpunkte geben 2 2 wie Sie zu Recht bemerkt haben, aber auch die Scheitelpunkte innerhalb der Kette. Dies liegt daran, dass für jeden der beiden φ Beine müssen Sie auswählen, wer mit dem externen Bein und wer mit einem anderen Kettenknoten verbunden ist. Also insgesamt hast du C = v ! × 2 v .

Berechnung der Dochtkontraktion in Theorie B:

Gleiche Begründung.

Weitere Informationen zu Symmetriefaktoren finden Sie in meiner Antwort Problem zum Verständnis des Symmetriefaktors in einem Feynman-Diagramm oder für die systematische Theorie unter Verwendung von Joyals Theorie kombinatorischer Arten in meinem Artikel "Feynman-Diagramme in der algebraischen Kombinatorik", aber das ist keine leichte Lektüre.

ja, n = v+2, es war ein Vorzeichenfehler. Die Zahl der φ Propagator muss ganzzahlig sein, aber Ihre Formel lässt halbzahlige Werte zu. zum Beispiel v=1 -> 5/2
@FrancescoS: Es gibt Brüche v / 2 in dem was du geschrieben hast. Ich hätte es getan, obwohl Sie sich dessen bewusst sind v muss sogar in Theorie A sein.
Entschuldigung, ich habe es nicht sehr gut erklärt. Tatsache ist, dass mich die Streuung von Skalarteilchen interessiert φ Feld. So kann ich gerade noch Propagatoren haben.
Es scheint, Sie interessieren sich für die Streuung von φ Felder nicht ϕ Felder.
Ja, es stimmt, ich habe meine Frage bearbeitet. Das ist der Grund, weil ich geschrieben habe v / 2 .
Das muss ich in diesem Limit beweisen λ = 3 G 2
Ich verstehe. Sie zerlegen den 4-wertigen Scheitelpunkt in Theorie B in zwei dreiwertige Scheitelpunkte mit einem Zwischenfeld ϕ . Der Faktor 3 liegt daran, dass es drei Möglichkeiten gibt, vier Elemente in zwei Paare zu gruppieren. Siehe zB mathoverflow.net/questions/149564/… für mehr über die Kombinatorik dieser Art von Hubbard-Stratonovic-Transformation.